Finding The Limit Of A 3D Function

Click For Summary
To find the limit of the function (X^2Y)/(X^2 + Y^4) as X and Y approach 0, various approaches were tested, including substituting different lines like y = x and x = y, all yielding a limit of zero. The user graphed the function, which also suggested a limit of zero. To rigorously prove this limit, transitioning to polar coordinates is recommended, using x = r*cos(θ) and y = r*sin(θ). This method can help analyze the limit more effectively as both r and θ approach their respective limits. Ultimately, using polar coordinates is a crucial step in confirming the limit.
Baumer8993
Messages
45
Reaction score
0

Homework Statement


Find the limit as X, and Y both approach 0 of (X2Y)/(X2 + Y4)


Homework Equations


The equation from above.


The Attempt at a Solution


I have been doing the technique of approaching from different lines such as y = x, or x = y,
x = 0, and y = 0. All of them give me a limit of zero, so that will not do. I graphed the function online, and it appears as if the function does have a limit of zero. How would I prove this? I think I need to do something with polar coordinates, but I am not sure how to do that.
 
Physics news on Phys.org
EDIT:
I'd go with SammyS's
 
Baumer8993 said:

Homework Statement


Find the limit as X, and Y both approach 0 of (X2Y)/(X2 + Y4)

Homework Equations


The equation from above.

The Attempt at a Solution


I have been doing the technique of approaching from different lines such as y = x, or x = y,
x = 0, and y = 0. All of them give me a limit of zero, so that will not do. I graphed the function online, and it appears as if the function does have a limit of zero. How would I prove this? I think I need to do something with polar coordinates, but I am not sure how to do that.
Yes. Change to polar coordinates.

x=r\cos(\theta)

y=r\sin(\theta)
 
Last edited:
Question: A clock's minute hand has length 4 and its hour hand has length 3. What is the distance between the tips at the moment when it is increasing most rapidly?(Putnam Exam Question) Answer: Making assumption that both the hands moves at constant angular velocities, the answer is ## \sqrt{7} .## But don't you think this assumption is somewhat doubtful and wrong?

Similar threads

  • · Replies 10 ·
Replies
10
Views
2K
  • · Replies 3 ·
Replies
3
Views
1K
  • · Replies 12 ·
Replies
12
Views
3K
  • · Replies 4 ·
Replies
4
Views
2K
  • · Replies 10 ·
Replies
10
Views
1K
  • · Replies 5 ·
Replies
5
Views
1K
  • · Replies 27 ·
Replies
27
Views
2K
  • · Replies 1 ·
Replies
1
Views
2K
  • · Replies 7 ·
Replies
7
Views
3K
  • · Replies 5 ·
Replies
5
Views
2K